Help finding the answer to DE using method of undetermined coefficients

Click For Summary
The discussion focuses on solving the differential equation y'' - 5y' = 2x^3 - 4x^2 - x + 6 using the method of undetermined coefficients. The user identifies the characteristic roots and constructs a particular solution, but encounters confusion when calculating the coefficients, resulting in unexpected values. It is suggested that the user may be making algebraic errors and should double-check their calculations. Emphasis is placed on using exact values rather than approximations to avoid further complications. The conversation highlights the importance of careful algebraic manipulation in solving differential equations.
lilmul123
Messages
39
Reaction score
0

Homework Statement



y'' - 5y' = 2x^3 - 4x^2 -x + 6




The Attempt at a Solution



I first found the m's on the left hand side. They are 0 and 5. I then found the m's on the right hand side. They are 0, 0, 0, and 0. I then wrote my interim equation:

y = C1 + C2e^5x + Ax^4 + Bx^3 + Cx^2 + Dx. Because there already is a 0 on the left hand size for m, then there are x's for each part of the equation on the right side.

This is where I'm confused. I have tried to find the coefficients by finding the first and second derivatives of Yp, but I get very odd numbers when I solve for the variables(B = 14/75 and C = 5.324 for example). Am I on the right track or did I screw up somewhere? We have never been presented with irrational coefficients before (as is the case with C), so I'm thinking I did something wrong somewhere...
 
Physics news on Phys.org
It sounds like you're on the right track. I think you're just making algebraic errors when solving for A, B, C, and D, so double-check your work for mistakes. If you post your actual work in solving for them, others and I can see if we spot any mistakes as well.
 
D'oh. You were right. Thank you!
 
I don't see anything wrong, but don't use approximations, since that will foul things up. Instead of 5.324, use the exact value.
 
There were no approximations, my algebra was just poor.
 
Question: A clock's minute hand has length 4 and its hour hand has length 3. What is the distance between the tips at the moment when it is increasing most rapidly?(Putnam Exam Question) Answer: Making assumption that both the hands moves at constant angular velocities, the answer is ## \sqrt{7} .## But don't you think this assumption is somewhat doubtful and wrong?

Similar threads

  • · Replies 15 ·
Replies
15
Views
3K
Replies
12
Views
2K
  • · Replies 3 ·
Replies
3
Views
1K
  • · Replies 14 ·
Replies
14
Views
3K
  • · Replies 1 ·
Replies
1
Views
2K
  • · Replies 2 ·
Replies
2
Views
1K
  • · Replies 14 ·
Replies
14
Views
2K
Replies
3
Views
2K
Replies
5
Views
3K
Replies
4
Views
3K